Last visit was: 25 Apr 2024, 20:13 It is currently 25 Apr 2024, 20:13

Close
GMAT Club Daily Prep
Thank you for using the timer - this advanced tool can estimate your performance and suggest more practice questions. We have subscribed you to Daily Prep Questions via email.

Customized
for You

we will pick new questions that match your level based on your Timer History

Track
Your Progress

every week, we’ll send you an estimated GMAT score based on your performance

Practice
Pays

we will pick new questions that match your level based on your Timer History
Not interested in getting valuable practice questions and articles delivered to your email? No problem, unsubscribe here.
Close
Request Expert Reply
Confirm Cancel
SORT BY:
Date
Tags:
Show Tags
Hide Tags
VP
VP
Joined: 30 Jan 2016
Posts: 1232
Own Kudos [?]: 4560 [0]
Given Kudos: 128
Send PM
Most Helpful Reply
Tutor
Joined: 16 Oct 2010
Posts: 14823
Own Kudos [?]: 64924 [1]
Given Kudos: 426
Location: Pune, India
Send PM
General Discussion
Manager
Manager
Joined: 10 Jun 2014
Posts: 70
Own Kudos [?]: 78 [0]
Given Kudos: 286
Location: India
Concentration: Operations, Finance
WE:Manufacturing and Production (Energy and Utilities)
Send PM
Verbal Forum Moderator
Joined: 08 Dec 2013
Status:Greatness begins beyond your comfort zone
Posts: 2101
Own Kudos [?]: 8810 [0]
Given Kudos: 171
Location: India
Concentration: General Management, Strategy
GPA: 3.2
WE:Information Technology (Consulting)
Send PM
Physician: In an experiment, 50 patients with chronic back pain were [#permalink]
Physician: In an experiment, 50 patients with chronic back pain were divided into two groups. Small magnets were applied to the backs of one group; the other group received no treatment. Most of the patients in the first group, but very few in the second group, reported a significant reduction in pain. This shows that magnetic fields are probably effective at relieving some back pain.

Which one of the following, if true, constitutes the logically strongest counter to the physician’s argument?
Conclusion- magnetic fields are probably effective at relieving some back pain

Pre-thinking- As with many causal relationships, the easiest way to weaken the conclusion is by recognizing an alternate cause. We want to think of a reason, other than the magnates, that the two groups differ. A key factor may be the fact that this was not a blind study. Participants knew if they were receiving the magnet treatment or no treatment by the nature of the study. After all, it would be hard not to notice a magnet on your back.


(A) A patient’s merely knowing that a treatment has been applied can lead to improvement in his or her condition.- Correct; the patient’s knowledge of the fact he or she is receiving treatment can impact subjective levels of pain
(B) Most physicians believe that medication relieves chronic back pain more effectively than magnets do.- incorrect; the causal argument was comparing magnet use to a complete lack of treatment. The argument does not attempt to compare types of treatment, but rather a type of treatment versus no treatment at all. The study was not attempting to determine what type of treatment is best to manage back pain, but rather could magnets work as a type of treatment to manage back pain.
(C) No other experiments have been done showing that magnetic fields reduce pain in any area other than the back.- Out of scope; the conclusion is specifically focused on “some back pain
(D) Some of the scientists who helped design the experiment believed even before the experiment that magnetic fields relieve back pain, but they were not directly involved in conducting the experiment.- incorrect; This answer choice attacks those who created the study. It sounds plausible that the developers’ beliefs could impact the results of the study. However, as we read through this answer choice, we realize that those researchers with a bias were not actually involved in carrying out the study.
(E) There was wide variation in the specific causes of the chronic back pain suffered by the patients in the experiment.- incorrect; It does not tell us, for example, that the first group consisted of patients with low back pain, while the second group consisted of patients with neck pain. It merely states that there were different types. Since it does not allow us to draw a new distinction between the two groups, it is not the correct answer choice.

Answer A

janadipesh wrote:
please explain the ans

Hi janadipesh,

Hope this helps! :)

Please read Magoosh blog in the following link for a better understanding

https://magoosh.com/gmat/2013/medical-t ... reasoning/
GMAT Club Bot
Physician: In an experiment, 50 patients with chronic back pain were [#permalink]
Moderators:
GMAT Club Verbal Expert
6921 posts
GMAT Club Verbal Expert
238 posts
CR Forum Moderator
832 posts

Powered by phpBB © phpBB Group | Emoji artwork provided by EmojiOne